You are on page 1of 38

Completion

Many airlines have been pushing the federal government for assurances of greater security
in case of terrorist attacks onboard flights. One of these airlines expects the improved
security will help increase revenue by $20 million a year, mostly from people who_____

Which of the following most logically completes the statement above?


A. are afraid of traveling by plane because of terrorist attacks
B. travel by train in the summer
C. spend more money while traveling
D. do not travel at all and spend less money on traveling
E. travel by plane when they are pressed for time
Increased security against terrorist attacks onboard flights is expected to lead to increased
revenue for airlines. Logically, this increase in revenue is likely to come from those
passengers who either avoid or minimize their current flight travels. Given the context,
such avoidance or minimization has to be linked to the likelihood of terrorist attacks
onboard the flights.

A. are afraid of traveling by plane because of terrorist attacks Correct answer. This
establishes a probable link between terrorist attacks and current avoidance or curtailing of
travel by air.

B. travel by train in the summer Incorrect. This has nothing to do with likelihood of
terrorist attacks onboard flights.

C. spend more money on traveling Incorrect. This has nothing to do with likelihood of
terrorist attacks onboard flights.

D. spend less money on traveling Incorrect. This has nothing to do with likelihood of
terrorist attacks onboard flights.

E. travel by plane when they are pressed for time Incorrect. This has nothing to do with
likelihood of terrorist attacks onboard flights.
While antibiotics have done inestimable good to humankind over the last seventy years,
there are several drawbacks to using antibiotics that, until recently, have been overlooked.
The human microbiome, which consists of the trillions of bacteria that reside in each
person’s body, is essential to good health. Specifically, the body contains and requires both
“good” and “bad” bacteria. It is when the proper equilibrium between the “good” bacteria
and the “bad” bacteria is disrupted that a number of health issues can emerge.
Nonetheless, antibiotics indiscriminately kill both the “good” and the “bad” bacteria, so each
course of antibiotics should be followed by a treatment that
__________________________________

Which of the following most logically completes the argument above?

A. bypasses the microbiome altogether


B. targets and eliminates only the “bad” bacteria
C. restores the correct balance between the “good” and “bad” bacteria
D. purges the body of the remaining antibiotics completely
E. reverses any effects from the antibiotics
A. bypasses the microbiome altogether
From passage it is understood that microbiome is inclusive of all bacteria on which
antibiotics acts/must act. hence bypassing microbiome does not make sense.
incorrect.
B. targets and eliminates only the “bad” bacteria
A treatment that targets only bad bacteria which is required by body will cause
further imbalance that is set in by antibiotics. hence incorrect.
C. restores the correct balance between the “good” and “bad” bacteria
Argument clearly states balance of good & bad bacteria is necessary. And antibiotics
kills both good & bad bacteria indiscriminately there by causing imbalance in
microbiome. Restoring this balance is the best course after antibiotics. CORRRECT
D. purges the body of the remaining antibiotics completely
Two issues with this:
1. The argument does not give enough information about whether there is
any remaining antibiotics.
2. Even if there is some remaining it may not have that much impact as it would be
less potent. - incorrect
E. reverses any effects from the antibiotics
we do not want to reverse effects of antibiotics, if we did then the positive use of
antibiotics is negated -incorrect
In many corporations, employees are being replaced by automated equipment in order to
save money. However, many workers who lose their jobs to automation will need
government assistance to survive, and the same corporations that are laying people off will
eventually pay for that assistance through increased taxes and unemployment insurance
payments.

The author is arguing that ________________.


(A) higher taxes and unemployment insurance payments will discourage corporations from
automating
(B) replacing people through automation to reduce production costs will result in increases
of other costs to corporations
(C) many workers who lose their jobs to automation will have to be retrained for new jobs
(D) corporations that are laying people off will eventually rehire many of them
(E) corporations will not save money by automating because people will be needed to run
the new machines
(A) higher taxes and unemployment insurance payments will discourage corporations from
automating. Author is not arguing , whether the whole scenario will discourage
the corporates or not.

(B) replacing people through automation to reduce production costs will result in increases
of other costs to corporations :- Perfect, this is what the author is arguing, An
indirect effect which will take place due to automation.

(C) many workers who lose their jobs to automation will have to be retrained for new jobs
:- Out of Scope , Argument does not say about future of workers who have or
will lose their job to automation.

(D) corporations that are laying people off will eventually rehire many of them :-
Irrelevant, Argument doesn't mention this anywhere.

(E) corporations will not save money by automating because people will be needed to run
the new machines :- Irrelevant, Argument is not concerned about the cost to
corporates.
Which of the following, if true, most logically completes the argument below?

Manufacturers are now required to make all cigarette lighters child-resistant by equipping
them with safety levers. But this change is unlikely to result in a significant reduction in the
number of fires caused by children playing with lighters, because children given the
opportunity can figure out how to work the safety levers and _______.

(A) the addition of the safety levers has made lighters more expensive than they were
before the requirement was instituted
(B) adults are more likely to leave child-resistant lighters than non-child-resistant lighters in
places that are accessible to children
(C) many of the fires started by young children are quickly detected and extinguished by
their parents
(D) unlike child-resistant lighters, lighters that are not child-resistant can be operated by
children as young as two years old
(E) approximately 5,000 fires per year have been attributed to children playing with lighters
before the safety levers were required
Answer A only talks about expense which does not relate to the likelihood of fires
Answer choice B says that not only will they be able to work the lighters but also will
have access to the lighters - therefore the safety levers will make little difference in the
number of fires.
Answer C does not address stopping children from starting fires, only that the fires can
be quickly extinguished
Answer D actually goes opposite of the argument, showing that there could be a
reduction because children under 2 could not start fires.
Answer E only tells us the number of fires, does not discuss whether children will be
able to start the fires.
Which of the following most logically completes the argument?

According to experts on shopping behaviour, more shoppers would shop at Jerrod’s


department store if they were offered the convenience of shopping carts. In fact, even if
the amount spent by these additional customers only just covered the cost of providing the
carts, providing carts would still probably increase Jerrod’s profits, since _______.

(A) the layout of Jerrod’s is open enough to accommodate shopping carts comfortably
(B) several department stores with shopping carts available to their customers
that compete with Jerrod’s have seen increased profits
(C) there are some potential customers who would not be enticed to shop at Jerrod’s by the
availability of shopping carts
(D) stores that make shopping carts available to customers usually have more revenues
(E) a customer with a shopping cart buys more, on average, than a customer without a cart
Which of the following most logically completes the argument?

According to experts on shopping behavior, more shoppers would shop at Jerrod’s department store
if they were offered the convenience of shopping carts. In fact, even if the amount spent by these
additional customers only just covered the cost of providing the carts, providing carts would still
probably increase Jerrod’s profits, since _______.

(A) the layout of Jerrod’s is open enough to accommodate shopping carts comfortably -- >Irrelevant
(Need a reason how shopping carts would increase more profit)
(B) several department stores that compete with Jerrod’s have begun to make shopping carts
available to their customers --> several department - Out of Scope
(C) there are some potential customers who would not be enticed to shop at Jerrod’s by the
availability of shopping carts --> potential customers who would not be enticed to shop - Out of Scope
(D) stores that make shopping carts available to customers usually have to hire people to retrieve
them from parking areas --> hire people to retrieve them from parking areas - Out of Scope
(E) a customer with a shopping cart buys more, on average, than a customer without a cart - Bingo
customer with a shopping cart buys more hence more profit
According to a study, after a week of high-altitude living, twenty men had slimmed down.
The men, middle-aged residents of low-altitude areas, had been taken to a research station
at 2,650 meters (8,694 feet) above sea level. They had unrestricted access to food and
were forbidden vigorous exercise, yet they lost an average of 1.5 kilograms (3.3 pounds)
during their one-week stay. Clearly, the lower availability of oxygen at higher altitudes, or
hypobaric hypoxia, can be said to have caused the weight loss, since __________.

Which of the following would, if true, most logically complete the argument?

A. a decrease in oxygen intake has been shown to depress appetite


B. the men all participated in the same kinds of exercise during their stay
C. the foods available to the men had fewer calories than the foods they usually ate
D. exercise at higher altitudes is more difficult than exercise at lower altitudes is
E. several weeks after returning home, the men gained weight
A. a decrease in oxygen intake has been shown to depress appetite-
Correct
B. the men all participated in the same kinds of exercise during
their stay- incorrect, we are only told that the men were forbidden
vigorous exercise
C. the foods available to the men had fewer calories than the foods
they usually ate- incorrect, they had unrestricted access to food
D. exercise at higher altitudes is more difficult than exercise at
lower altitudes is- incorrect, this does not strengthen our claim.
Also, we do not even know whether these men excercised
E. several weeks after returning home, the men still weighed less
than they had before the study - irrelevant, what happens after
returning home is not relevant
For the past several years, a certain technology has been widely used to transmit data
among networked computers. Recently two data transmission companies, Aptron and
Gammatech, have each developed separate systems that allow network data transmission at
rates ten times faster than the current technology allows. Although the systems are similarly
priced and are equally easy to use, Aptron's product is likely to dominate the market,
because __________.

(A) Aptron has been in the business of designing data transmission systems for several years
more than Gammatech has
(B) the number of small businesses that need computer networking systems provided by
Aptron is likely to double over the next few years
(C) it is much more likely that Aptron 's system will be expandable to meet future needs if
they come
(D) unlike Gammatech, Aptron develop computers in addition to data transmission systems
(E) it is easier for users of the current data transmission technology to switch to Aptron's
product than to Gammatech's
For the past several years, a certain technology has been widely used to transmit data among
networked computers. Recently two data transmission companies, Aptron and Gammatech, have each
developed separate systems that allow network data transmission at rates ten times faster than the
current technology allows. Although the systems are similarly priced and are equally easy to use,
Aptron's product is likely to dominate the market, because __________.

(A) Aptron has been in the business of designing data transmission systems for several years more
than Gammatech has
(B) the number of small businesses that need computer networking systems provided by Aptron is
likely to double over the next few years
(C) it is much more likely that Aptron 's system will be expandable to meet future needs if they come
(D) unlike Gammatech, Aptron develop computers as well in addition to data transmission systems
(E) it is easier for users of the current data transmission technology to switch to Aptron's product than
to Gammatech's
Error Analysis:
1) Good, Should increase the appeal of A over G due to a possible role loyalty
coming in to the picture but not enough
2) Good, would increase demand for A but not enough
3) Good, would increase the appeal of A over G since it is more pertinent in the
future but we are in present!
4) Good to make computers but Irrelevant
5) Here's our differentiating factor that ranks A over G, CORRECT
Arnold: I was recently denied a seat on an airline flight for which I had a confirmed reservation,
because the airline had overbooked the flight. I was offered to fly on the next available flight, which did
not depart until two hours later. Further, the second flight was cancelled at the last minute due to bad
weather, I missed an important business meeting. Therefore, the airline should still pay me
compensation for the loss.

A principle that, if established, justifies that an airline is morally obligated to compensate a passenger
who has been denied a seat on a flight for which the passenger has confirmed reservations________

(A) if the only reason the passenger is offered to take a later flight is that the airline overbooked the
flight
(B) only if there is a reason the passenger is offered to take a later flight other than the original flight's
being cancelled due to bad weather
(C) only if the passenger would not have been offered to take a later flight had the airline not
overbooked the original flight
(D) even if the only reason the passenger is offered to take a later flight were at the original flight is
canceled due to bad weather
(E) even if the passenger would still have been offered to take a later flight had the airline not
overbooked the original flight
Arnold: I was recently denied a seat on an airline flight for which I had a confirmed reservation,
because the airline had overbooked the flight. I was offered to fly on the next available flight, which did
not depart until two hours later. Further, the second flight was cancelled at the last minute due to bad
weather, I missed an important business meeting. Therefore, the airline should still pay me
compensation for the loss.

A principle that, if established, justifies that an airline is morally obligated to compensate a passenger
who has been denied a seat on a flight for which the passenger has confirmed reservations________

(A) if the only reason the passenger is offered to take a later flight is that the airline overbooked the
flight
(B) only if there is a reason the passenger is offered to take a later flight other than the original flight's
being cancelled due to bad weather
(C) only if the passenger would not have been offered to take a later flight had the airline
not overbooked the original flight
(D) even if the only reason the passenger is offered to take a later flight were at the original flight is
canceled due to bad weather
(E) even if the passenger would still have been offered to take a later flight had the airline not
overbooked the original flight
Extraaaa
Some batches of polio vaccine used around 1960 were contaminated with SV40, a virus
that in monkeys causes various cancers. Some researchers now claim that this
contamination caused some cases of a certain cancer in humans, mesothelioma. This
claim is not undercut by the fact that a very careful survey made in the 1960s of people
who had received the contaminated vaccine found no elevated incidence of any cancer,
since __________.

(A) most cases of mesothelioma are caused by exposure to asbestos

(B) in some countries, there was no contamination of the vaccine

(C) SV40 is widely used in laboratories to produce cancers in animals

(D) mesotheliomas take several decades to develop

(E) mesothelioma was somewhat less common in 1960 than it is now


Premises:
Some polio vaccines in 1960s were contaminated by SV40.
Researchers claim that SV40 can cause mesothelioma.

Question stem: This claim is not undercut by the fact that a very careful survey made
in the 1960s of people who had received the contaminated vaccine found no elevated
incidence of any cancer, since___________.

Normally, if one would say "a very careful survey made in the 1960s of people who
had received the contaminated vaccine found no elevated incidence of any cancer",
this would weaken the claim of the researchers. The researchers say that SV40 could
cause mesothelioma but since no elevated incidence was found, we would doubt the
claim of the researchers.
But we need to find the reason this claim is not put in doubt by "no elevated
incidence found".

Look at option (D): mesotheliomas take several decades to develop

The contaminated vaccine is given in 1960s. The survey was conducted in 1960s
itself. If mesothelioma takes several decades to develop, we will not see an elevated
incidence in the 60s decade itself. We will see an elevated incidence in later decades.
Hence this option nicely completes the sentence.

All other options are irrelevant.


Opponents to the mayor’s plan for express bus lanes on the city’s major commuter arteries objected that people
could not be lured out of their automobiles in that way. The opponents were proved wrong; following
implementation of the plan, bus ridership rose dramatically, and there was a corresponding drop in automobile
traffic. Nonetheless, the plan failed to achieve its stated objective of reducing average commuting time.

Which of the following would be the most logical continuation of this argument?

(A) The plan’s opponents failed to realize that many people would take advantage of improved bus
transportation.

(B) Unfortunately, politically attractive solutions do not always get results.

(C) The number of people a vehicle can transport varies directly with the size of the passenger compartment of
the vehicle.

(D) Opponents cited an independent survey of city commuters showing that before the plan’s adoption only one
out of every seven used commuter bus lines.

(E) With the express lanes closed to private automobile traffic, the remaining cars were forced to use too few
lanes and this created gigantic traffic tie-ups.
Premise states that: bus express lanes have helped the dramatic
rise in ridership, and a corresponding dip in automobile ensued.

However, realize that if that's the case, either one of two things
has to happen.
1) Highway builds new lanes
2) one of the existing lanes is now the bus express lane, thus
resulting in one fewer lane for automobiles.

E describes this exactly.


With the rise of the internet, social media, and mobile phones, spreading news has become very easy. A new
crowd of ‘citizen journalists’ – people sharing pictures of traffic jams, blogging about local events, providing their
individual opinions on websites – can be seen everywhere. However, the idea that professional journalism will
soon decline is premature at best because _____________.

Which of the following best completes the passage?

A. many people acting as ‘citizen journalists’ would not want to take journalism as a profession
B. many people acting as ‘citizen journalists’ are only interested in certain types of news
C. professional journalists present the news in a much more polished manner
D. professional journalists provide insights that citizen journalists cannot
E. professional journalists are dedicated to their job while citizen journalists are less reliable in news reporting
A. many people acting as ‘citizen journalists’ would not want to take journalism as a profession
Incorrect. Citizen journalists are "citizen" journalists because they do not take journalism as a profession. That is
the distinction between "citizen journalists" and "professional journalists".

B. many people acting as ‘citizen journalists’ are only interested in certain types of news
This doesn't tell you why professional journalism will not decline. If people are interested in reporting only certain
types of news, one would expect that professional journalists will become less significant in those fields. Say, as
per the argument, need of professional journalists to cover traffic situations should decrease. So even if option (B)
is true, we would expect professional journalism to decline in at least some specific fields.

C. professional journalists present the news in a much more polished manner


So, the quality of professional journalists' news is higher. Sounds reasonable.

D. professional journalists provide insights that citizen journalists cannot


Again, the quality of professional journalists' news is higher. Sounds reasonable. Mind you, we are not here to
judge what defines quality. We may feel that "polished" is superficial and "insights" is what really matters but
people's opinion may be different. We cannot take a call on this based on our own opinion. CR expects us to
evaluate other people's opinions, not give our own.

E. professional journalists are dedicated to their job while citizen journalists are less reliable in news reporting
Does "citizen journalists are less reliable in news reporting" mean that "professional journalists" are more reliable?
Perhaps. But since I have to imply that, I will go with another option which clearly states that quality of
professional journalists' news is higher.
"professional journalists are dedicated to their job" doesn't add anything to the argument. Does it bother the
reader whether the person giving news is dedicated or working part time? No. As long as the news I get is good
quality, it doesn't matter.
Twenty percent of the stores in Morganville's downtown shopping district will fail within five years because they
will be competing directly with the SaveMart discount department store newly opened in East Morganville. The
downtown shopping district has lost business at this rate before and has always completely rebounded.
Confidence that it will rebound again from the losses it is now about to suffer is ill founded, however, because
________ .

(A) the stores like to be put out of business by direct competition from SaveMart are the downtown shopping
district's anchor stores, on whose ability to draw shoppers many of the other downtown stores depend.

(B) the bus line that has long connected the downtown area of Morganville with EastMorganville has a tradition of
carrying shoppers who reside in EastMorganville into downtown Morganville to shop.

(C) when the downtown shopping district has rebounded before, the business premises of a failed business were
typically taken over by a business of same kind as had been there before

(D) SaveMart's business plan for the EastMorgalvill store is based on earning low profits, if any, during the first
five years of the store's existence.

(E) it is conceivable that the downtown shopping district could shrink substantially without collapsing altogether
(A) the stores like to be put out of business by direct competition from SaveMart are the downtown shopping district's anchor
stores, on whose ability to draw shoppers many of the other downtown stores depend.
If stores to be put out of business are the ANCHOR stores that other depends on, then it will be hard to recuperate.
This is a keeper. Let's keep this for now.

(B) the bus line that has long connected the downtown area of Morganville with EastMorganville has a tradition of carrying shoppers
who reside in East Morganville into downtown Morganville to shop.
What if majority of the shoppers are non-bus riders... These majority will drive the possibility of recovery for the
downtown district... WE have to assume a little more to make this work... And BUS is accessible to both the
downtown and the new SaveMart... competition is not helped if both shops are equally accessible...

(C) when the downtown shopping district has rebounded before, the business premises of a failed business were typically taken
over by a business of same kind as had been there before
This describes what happened in the past when a business failed. This has no bearing or explanation on whether to
recover in the new case at hand is possible. OUT!

(D) SaveMart's business plan for the EastMorgalvill store is based on earning low profits, if any, during the first five years of the
store's existence.
This explains the cheap price of the SaveMart, as mentioned already in the argument "discount department
stores".... No additional info on whether the shopping district will be able to recover... UNLIKE in A.... A so far is a
contender.

(E) it is conceivable that the downtown shopping district could shrink substantially without collapsing altogether
Shrinking without collapsing altogether.. shows that there is a chance... This is opposite of what we need...
Answer: A
Within the earth's core, which is iron, pressure increases with depth. Because the
temperature at which iron melts increases with pressure, the inner core is solid and the
outer core is molten. Physicists can determine the melting temperature of iron at any
given pressure and the pressure for any given depth in the earth. Therefore, the actual
temperature at the boundary of the earth's outer and inner cores—the melting
temperature of iron there—can be determined, since_______.

(A) the depth beneath the earth's surface of the boundary between the outer and inner
cores is known
(B) some of the heat from the earth's core flows to the surface of the earth
(C) pressures within the earth's outer core are much greater than pressures above the
outer core
(D) nowhere in the earth's core can the temperature be measured directly
(E) the temperatures within the earth's inner core are higher than in the outer core
Argument:- Within the earth's core, which is iron, pressure increases with depth. Because
the temperature at which iron melts increases with pressure, the inner core is solid and
the outer core is molten. Physicists can determine the melting temperature of iron at any
given pressure and the pressure for any given depth in the earth. Therefore, the actual
temperature at the boundary of the earth's outer and inner cores--the melting
temperature of iron there--can be determined, since_______.

The correct answer choice is "A" and my explanation as follows:

Option A states "a) the depth beneath the earth's surface of the boundary between the
outer and inner cores is known"
The highlighted portion above in the argument states that for a "given" pressure / depth it
is possible to determine temperature therefore the assumption that would logically
complete this argument is that either or both of these parameters are "given" or in other
words "known".
When the products of several competing suppliers are perceived by consumers to be essentially the
same, classical economics predicts that price competition will reduce prices to the same minimal
levels and all suppliers’ profits to the same minimal levels. Therefore, if classical economics is true,
and given suppliers’ desire to make as much profit as possible, it should be expected that
____________

(A) in a crowded market widely differing prices will be charged for products that are essentially the
same as each other

(B) as a market becomes less crowded as suppliers leave, the profits of the remaining suppliers will
tend to decrease

(C) each supplier in a crowded market will try to convince consumers that its product differs
significantly from its competitors’ products.

(D) when consumers are unable to distinguish the products in a crowded market, consumers will
judge that the higher-priced products are of higher quality

(E) suppliers in crowded markets will have more incentive to reduce prices and thus increase sales
than to introduce innovations that would distinguish their product from their competitors’ products
Reading the passage ; discussion about the classic theory and
product cost. When you carefully read the passage you will find
the that the customers are perceiving the products to be the
same; which gives you a hint of what the correct option might
based on. Then while scrolling through the options C, D, E seems
to be the answers. But wait, E discuss about innovation; D
discusses about the product quality both of which are out of
scope. Remaining is C, But wait, as A also seems correct which
states about the different price tags at lower levels ? But if you
remember the first point that the consumer perceived the
products same. So what going to make them convinced to buy
something which is perceived as same. Yes if they are convinced
that the product they are purchasing is different. Hence and-C
Appendicitis (inflammation of the appendix) is potentially fatal; consequently, patients with symptoms strongly
suggesting appendicitis almost always have their appendix removed. The appropriate surgery is low-risk but
performed unnecessarily in about 20 percent of all cases. A newly developed internal scan for appendicitis is
highly accurate, producing two misdiagnoses for every 98 correct diagnoses. Clearly, using this test, doctors can
largely avoid unnecessary removals of the appendix without, however, performing any fewer necessary ones than
before, since __________.

(A) the patients who are correctly diagnosed with this test as not having appendicitis invariably have medical
conditions that are much less serious than appendicitis

(B) the misdiagnoses produced by this test are always instances of attributing appendicitis to someone who does
not, in fact, have it

(C) all of the patients who are diagnosed with this test as having appendicitis do, in fact, have appendicitis

(D) every patient who is diagnosed with this test as having appendicitis has more than one of the symptoms
generally associated with appendicitis

(E) the only patients who are misdiagnosed using this test are patients who lack one or more of the symptoms
that are generally associated with appendicitis
Should be (B)

Say 80 out of 100 cases were correctly diagnosed per the


previous surgery. Using the new scan, how would you ensure that
the 80 surgeries that were previously diagnosed correctly will
continue to be accurately detected? This is possible only when
the new scan detects 98 surgeries that are correct (AND
INCLUDES the 80 correct ones from the earlier surgery) and
perhaps misdiagnoses 2 (that are essentially false positives). It is
essential that these 2 be false positives, as the new scan should
not leave out any real cases that were correctly identified before.
Using broad-spectrum weed killers on weeds that are competing with crops for sunlight, water, and nutrients
presents a difficulty: how to keep the crop from being killed along with the weeds. For at least some food crops,
specially treated seed that produces plants resistant to weed killers is under development. This resistance wears
off as the plants mature. Therefore, the special seed treatment will be especially useful for plants that
_____________ .

Which of the following most logically completes the argument below?

(A) produce their crop over an extended period of time, as summer squash does
(B) produce large seeds that are easy to treat individually, as corn and beans do
(C) provide, as they approach maturity, shade dense enough to keep weeds from growing
(D) are typically grown in large tracts devoted to a single crop
(E) are cultivated specifically for the seed they produce rather than for their leaves or roots
..... weeds that are competing with crops for sunlight, water, and
nutrients ....

Special seeds are weed resistant but the Resistance goes off one
they mature,
(C) says that
-provide, as they approach maturity, shade dense enough to keep
weeds from growing
means they will become self sufficient when grow up and hence
can protect themselves even if the resistance goes off which was
there by the virtue og the seeds they grew on
A new machine for harvesting corn will allow rows to be planted only fifteen inches apart, instead
of the usual thirty inches. Corn planted this closely will produce lower yields per plant.
Nevertheless, the new machine will allow corn growers to double their profits per acre because
________.

(A) with the closer spacing of the rows, the growing corn plants will quickly form a dense canopy of
leaves, which will, by shading the ground, minimize the need for costly weed control and irrigation
(B) with the closer spacing of the rows, corn plants will be forced to grow taller because of
increased competition for sunlight from neighboring corn plants
(C) with the larger number of plants growing per acre, more fertilizer will be required
(D) with the spacing between rows cut by half, the number of plants grown per acre will almost
double
(E) with the closer spacing of the rows, the acreage on which corn is planted will be utilized much
more intensively than it was before, requiring more frequent fallow years in which corn fields are
left unplanted
(A) with the closer spacing of the rows, the growing corn plants will quickly form a dense canopy of leaves, which
will, by shading the ground, minimize the need for costly weed control and irrigation
This option is in line with our prethought strengthener. Hence this is the right answer.

(B) with the closer spacing of the rows, corn plants will be forced to grow taller because of increased competition
for sunlight from neighboring corn plants
This is not a strengthener because taller rows do not mean more yield. Therefore, it is not the correct answer.

(C) with the larger number of plants growing per acre, more fertilizer will be required
This option is a weakener because it talks about increasing expenditure on the corn fields. Therefore it is a wrong
answer choice.

(D) with the spacing between rows cut by half, the number of plants grown per acre will almost double
This option cannot be the correct answer because this information is already given in the passage. It provides no
new information. Remember – a strengthener must bring in new information we cannot infer from the passage. We
are already given that instead of 30 inches apart, now we have the gap between successive corn rows as only 15
inches. Which essentially means pretty much double the number of corn rows, and so, pretty much double the
number of plants grown. So, option D is not new information.

(E) with the closer spacing of the rows, the acreage on which corn is planted will be utilized much more intensively
than it was before, requiring more frequent fallow years in which corn fields are left unplanted
This option also acts as a weakener because it talks about frequent years in which farming corn cannot be done.
Hence this is an incorrect answer.
Which of the following most logically completes the passage below?
Heavy rains during Centralia’s corn planting season prevented some farmers there from
planting corn. It is now the planting season for soybeans, another of Centralia’s principal
crops, and those fields originally intended for corn are dry enough for planting.
Nonetheless, even though soybean prices are unusually high at present, the farmers will
leave most of these fields empty rather than plant them with soybeans, since ‗‗‗‗‗‗.

A. the extensive rains have led to an increase in the price of corn


B. some Centralian farmers anticipate serious financial losses due to the extremely wet
spring planting season
C. chemicals that were used to prepare the fields for corn planting would stunt the growth
of soybeans
D. the majority of Centralia’s corn farmers were able to plant corn as they had intended,
despite the wet planting season
E. many Centralian farmers grow both corn and soybeans
A. the extensive rains have led to an increase in the price of corn
The planting season for corn is over. price increase must be due to limited supply. It is unlikely
that increase in price can lead farmers to plant corn in a non season. Hence not the reason.

B. some Centralian farmers anticipate serious financial losses due to the extremely wet spring
planting season
irrelevant... We dont know which season is.. spring planting season? we know about corn
planting season and soybean planting seasons. Not the reason.

C. chemicals that were used to prepare the fields for corn planting would stunt the growth of
soybeans
If the growth of soybean is hampered due to chemicals, farmers would again suffer a loss.
Hence they might not plant the fields with soybeans. Possible reason.

D. the majority of Centralia’s corn farmers were able to plant corn as they had intended, despite
the wet planting season
Totally irrelevant. Farmers were able to plant corn as they intended. But it doesnt help in
explaining the discrepancy. Not the reason.

E. many Centralian farmers grow both corn and soybeans


Again ,irrelavant. Yeah we know that - both are principal crops in centralia. But it doesnt help in
explaining the discrepancy.Not the reason.

C presents only possible reason.

You might also like